285. Nice and easy inequalities. For example, IMAC - 2009.

by Virgil Nicula, Jun 13, 2011, 1:42 PM

IMAC - 2009. Prove that for the sides of triangle $ABC$ exists the inequality $\frac{a^3}{b+c-a} + \frac{b^3}{a+c-b} + \frac{c^3}{a+b-c} \geq a^2 + b^2 + c^2$ .

Proof 1. $\sum\, \frac {a^3}{b+c-a}=\sum\, \frac {a^4}{a(b+c-a)}\, \stackrel{C.B.S.}{\ge}\, \frac {\left(a^2+b^2+c^2\right)^2}{2(ab+bc+ca)-(a^2+b^2+c^2)}$ thus remaining to show that

$\frac {\left(a^2+b^2+c^2\right)^2}{2(ab+bc+ca)-(a^2+b^2+c^2)}\, \ge\, a^2+b^2+c^2\iff a^2+b^2+c^2\ge ab+bc+ca$ , which is obviously.

Proof 2. Since $\sum (b+c-a)=\sum a$ and $a^2\le b^2\le c^2\iff$ $a\le b\le c\iff$ $\frac {1}{b+c-a}\le\frac {1}{c+a-b}\le\frac {1}{a+b-c}\iff$ $a^3\le b^3\le c^3$ .

Can apply the Chebyshev's inequality so that $:\ \left\{\begin{array}{c}
\sum a^2\cdot\sum a\le 3\cdot\sum a^3\\\\
9\le \sum (b+c-a)\cdot\sum \frac {1}{b+c-a}\\\\
\sum a^3\cdot \sum\frac {1}{b+c-a}\le 3\cdot\sum\frac {a^3}{b+c-a}\end{array}\right\|\ \bigodot\ \implies\ \sum a^2\le \sum\frac {a^3}{b+c-a}$ .


Similar proposed problem. Prove that $\sum\frac {a^3}{(s-b)(s-c)}\ge \frac {24\cdot S}{2R-r}$ and $\sum\frac {1}{s-a}\ge \frac {s}{r(2R-r)}$ , where $S=[ABC]$ and $2s=a+b+c$ .

Proof. I"ll use the well-known identity $\sum a(s-b)(s-c)=2sr(2R-r)$ . Thus,$\sum\frac {a^3}{(s-b)(s-c)}=$ $\sum\frac {\left(a^2\right)^2}{a(s-b)(s-c)}\stackrel{(C.B.S.)}{\ge}$

$ \frac {\left(\sum a^2\right)^2}{\sum a(s-b)(s-c)}=$ $\frac {\left(\sum a^2\right)^2}{2sr(2R-r)}\stackrel{(\mathrm{Weitzenbock})}{\ge}$ $\frac {48\cdot S^2}{2\cdot S(2R-r)}=$ $\frac {24\cdot S}{2R-r}$ . I"ll proceed analogously $:\ \sum\frac {1}{s-a}=$ $\sum \frac {(s-b)+(s-c)}{2(s-b)(s-c)}=$

$\sum \frac {a}{2(s-b)(s-c)}=$ $\sum\frac {a^2}{2a(s-b)(s-c)}\stackrel{(C.B.S.)}{\ge}$ $\frac {(\sum a)^2}{2\cdot \sum a(s-b)(s-c)}=$ $\frac {2s^2}{2sr(2R-r)}=\frac {s}{r(2R-r)}$ .

Otherwise
.
$\sum\frac {1}{s-a}=$ $ \frac {\sum (s-b)(s-c)}{\prod (s-a)}=$ $\frac {r(4R+r)}{sr^2}=$ $\frac {4R+r}{sr}$ . Therefore, $\sum\frac {1}{s-a}\ge \frac {s}{r(2R-r)}\iff$ $(4R+r)(2R-r)\ge s^2$ .

Observe that $s^2\stackrel{\mathrm{(Gerretsen)}}{\le} 4R^2+4Rr+3r^2$ and $4R^2+4Rr+3r^2\le (4R+r)(2R-r)\iff$ $2R^2-3Rr-2r^2\ge 0\iff$

$(R-2r)(2R+r)\ge 0$ what is obviously. Therefore obtain that $s^2\le (4R+r)(2R-r)$ . In conclusion and $\sum\frac {1}{s-a}\ge \frac {s}{r(2R-r)}$ .



PP2. Prove that $\left\{a,b,c\right\}\subset (0,1]\ \implies\ a+b+c+ \frac{1}{abc} \ge\frac{1}{a} + \frac{1}{b} + \frac{1}{c} +abc$ .

Proof. Denote $\left\{x,y,z\right\}\subset (0,1]$ so that $ax=by=cz=abc\equiv p$ . Observe that $xyz=p^2$ and our inequality

becomes $p\cdot\sum \frac 1x+\frac 1p\ge\frac 1p\cdot\sum x+p\iff$ $\frac 1p\cdot \sum yz+\frac 1p\ge \frac 1p\cdot\sum x+\frac 1p\cdot xyz\iff$ $1-\sum x\ +$

$\sum yz-xyz\ge 0\iff$ $(1-x)(1-y)(1-z)\ge 0\iff$ $(1-bc)(1-ca)(1-ab)\ge 0$ , what is truly.


An easy and nice extension. Prove that $(\forall )\ k\in\overline{1,n}\ ,\ x_k\in (0,1]\implies \sum_{k=1}^nx_k+$ $\frac {1}{x_1x_2\ldots x_{n-1}x_n}\ \ge\ \sum _{k=1}^n\frac {1}{x_k}+$ $x_1x_2\ldots x_{n-1}x_n$ .

Proof. Denote $f(x)=\frac 1x-x$ , where $x\in (0,1]$ . Prove easily that $f(ab)\ge f(a)+f(b)\iff$ $(1-a)(1-b)(1-ab)\ge 0$ .

Therefore, $\left\{\begin{array}{c}
f(x_1x_2)\ge f(x_1)+f(x_2)\\\\
f(x_1x_2x_3)\ge f(x_1x_2)+f(x_3)\\\\
f(x_1x_2x_3x_4)\ge f(x_1x_2x_3)+f(x_4)\\\
...............................................................\\\
f(x_1x_2\ldots x_{n-1}x_n)\ge f(x_1x_2\ldots x_{n-1})+f(x_n)\end{array}\right|\ \bigoplus\ \implies\ f(x_1x_2\ldots x_n)$ $\ge \sum_{k=1}^nf(x_k)\iff$

$\frac {1}{x_1x_2\ldots x_n}-x_1x_2\ldots x_n\ge \sum_{k=1}^n\left(\frac {1}{x_k}-x_k\right)\iff$ $\sum_{k=1}^n x_k+\frac {1}{x_1x_2\ldots x_n}\ge \sum_{k=1}^n\frac {1}{x_k}+x_1x_2\ldots x_n$ .



PP3. Prove that for any point $P$ which is interior to $\triangle ABC$ there is the relation $\frac {PA}{bc}+\frac {PB}{ca}+\frac {PC}{ab}\ge\frac 1R$ .

Proof. Observe that $PA+\delta_{BC}(P)\ge h_a$ a.s.o. and $\sum a\cdot\delta_{BC}(P) =2S$ . Therefore,

$\sum a\cdot\left[PA+\delta_{BC}(P)\right]\ge\sum ah_a=6S\implies$ $\sum a\cdot PA\ge 4S\implies$ $\sum\frac {PA}{bc}\ge \frac {4S}{abc}=\frac 1R$ .



PP4. Find the minimum value of $E(x,y)\equiv \cos x+\cos y+2\cos (x+y)$ .

Proof. Denote $\left\{\begin{array}{c}
x=a+b\\\
y=a-b\end{array}\right|$ . Observe that $E(x,y)=\cos (a+b)+\cos (a-b)+2\cos 2a=$ $2\cos a\cos b+2\left(2\cos^2a-1\right)=$

$4\underline{\cos ^2a}+2\cos b\cdot\underline{\cos a}-2=$ $\left(2\cos a+\frac 12\cdot\cos b\right)^2+\frac 14\cdot\sin^2 b-\frac 14-2=$ $\frac 14\cdot\left[\left(4\cos a+\cos b\right)^2+\sin^2b-9\right]\ge -\frac 94$ .

We have equality if and only if $4\cos a+\cos b=0\ \ \wedge\ \ \sin b=0$ , i.e. $\sin b=0\ \ \wedge\ \ \cos a=\frac {\epsilon}{4}$ , where $\epsilon^2=1$ .



PP5. Let $ABC$ be a triangle so that $b\ge c$ . Prove that $\frac ba+\frac {a}{b+c}\ge\sqrt {2+\frac {b^2-c^2}{a^2}}\ (*)$ with equality

iff $A=2C$ . Remark. If $B=90^{\circ}$ , then our inequality becomes $\frac ba+\frac {a}{b+c}\ge\sqrt 3$ with equality iff $A=60^{\circ}$
.

Proof 1. If $b=c$ , then the inequality $(*)$ becomes $\frac ba+\frac {a}{2b}\ge \sqrt 2\iff$ $(a-b\sqrt 2)^2\ge 0$ , what is true. We have equality iff $a=b\sqrt 2$ , i.e. $\sin \frac A2=\frac {\sqrt 2}{2}\iff$

$A=2C$ . Suppose $b>c$ and denote $b^2-c^2=\lambda a^2$ , i.e. $b^2=c^2+\lambda a^2$ , where $\lambda >0$ . Inequality $(*)$ becomes $\frac ba+\frac {b-c}{\lambda a}\ge \sqrt {2+\lambda}\iff$

$(1+\lambda )b\ge c+\lambda a\cdot\sqrt {2+\lambda }\iff$ $(\lambda +1)^2\left(\lambda a^2+c^2\right)\ge $ $c^2+2ac\lambda \sqrt {2+\lambda}+\lambda^2a^2(2+\lambda )$ $\iff$ $a^2-2ac\sqrt{2+\lambda }+(2+\lambda )c^2\ge 0\iff$

$\left(a-c\sqrt {2+\lambda }\right)^2\ge 0$ , what is true. We have equality iff $a=c\cdot\sqrt {2+\lambda}$ , i.e. $\left|a^2-c^2\right|=bc\iff$ $|A-B|=C\iff A=2B$ .

Remark. If $\lambda =1$ , i.e. $b^2=a^2+c^2 \iff$ $B=90^{\circ}$ , then $(*)$ becomes $\frac ba+\frac {a}{b+c}\ge \sqrt 3$ , with equality iff $A=60^{\circ}$ .

Proof 2. By squaring both sides of the given inequality one obtains: $\frac {b^2}{a^2}+\frac {a^2}{(b+c)^2}+\frac {2b}{b+c}\, \ge\, 2+\frac {b^2}{a^2}-\frac {c^2}{a^2}$ $\iff$ $\frac {a^2}{(b+c)^2}+\frac {c^2}{a^2}\, \ge\, \frac {2c}{b+c}$ , which

is trivial by AM-GM inequality. Equality occurs iff $\frac a{b+c}=\frac ca\iff$ $a^2=bc+c^2\iff$ $ a^2+b^2-c^2=$ $b^2+bc\iff 2ab\cos C=b^2+bc\iff$

$2a\cos C=b+c\iff$ $2a\cos C=\frac {a^2}c\iff$ $2c\cos C=a\iff 2\sin C\cos C=$ $\sin A\iff\sin 2C=\sin A\iff A=2C$ .



PP6. Let $P$ be a point inside the $\triangle ABC$ . Denote the distances $x$ , $y$ and $z$ from $P$ to the sides of $\triangle ABC$. Prove

that $R \leq \frac{a^2+b^2+c^2}{18\cdot\sqrt [3]{xyz}}$ , where R is the circumradius of the triangle. When does equality occur ? (Taiwan TST 2005)


Proof. $\sum ax=2S\implies$ $3\cdot \sqrt[3]{abcxyz}\le 2S\ (*)$ . So, $3\cdot\sqrt [3]{a^2b^2c^2}\le \sum a^2\implies$ $12RS=3\cdot a^2b^2c^2\le\sqrt [3]{abc}\cdot\sum a^2\implies$

$12RS\cdot \sqrt [3]{xyz}\le \left(a^2+b^2+c^2\right)\cdot \sqrt[3]{abcxyz}\stackrel{(*)}{\le} \left(a^2+b^2+c^2\right)\cdot\frac {2S}{3}\implies$ $18R\cdot\sqrt [3]{xyz}\le a^2+b^2+c^2$ .


PP7. Prove that $\{x,y,z\}\subset\mathbb R\ \implies\ \sqrt{x^2-xy+y^2}+\sqrt{y^2-yz+z^2}\ge \sqrt{x^2+xz+z^2}$ .

Proof 1. Minkowski's inequality : $\sqrt {{{\left( {y - \frac{x}{2}} \right)}^2} + {{\left( {\frac{{x\sqrt 3 }}{2}} \right)}^2}}  + \sqrt {{{\left( {\frac{z}{2} - y} \right)}^2} + {{\left( {\frac{{z\sqrt 3 }}{2}} \right)}^2}}$ $ \ge \sqrt {{{\left( {\frac{{x - z}}{2}} \right)}^2} + {{\left[ {\frac{{\left( {x + z} \right)\sqrt 3 }}{2}} \right]}^2}}  = \sqrt {{x^2} + xz + {z^2}}$ .

Proof 2. Let $\triangle VBA$ , $\triangle VBC$ where $VB$ separates $A$ , $C$ and $\left\{\begin{array}{ccc}
VA=x & VB=y & VC=z\\\\
m\left(\widehat{AVB}\right)=60^{\circ} & m\left(\widehat{CVB}\right)=60^{\circ} & m\left(\widehat{AVC}\right)=120^{\circ}\end{array}\right\|$ .

Observe that $\left\{\begin{array}{c}
AB^2=x^2-xy+y^2\\\\
CB^2=z^2-zy+y^2\\\\
AC^2=x^2+xz+z^2\end{array}\right\|$ and our inequality is equivalently with $AB+CB\ge AC$ , what is truly. We have equality iff $B\in (AC)\ \iff$

in $\triangle AVC$ the edge $[VB]$ is the $V$-bisector in $\triangle AVC\iff$ $\frac {BA}{BC}=\frac {VA}{VC}\iff$ $\frac {\sqrt{y^2-yx+x^2}}{y^2-yz+z^2}=\frac xz\iff$ $z^2\left(y^2-yx+x^2\right)=x^2\left(y^2-yz+z^2\right)\iff$

$y^2\left(x^2-z^2\right)=xyz(x-z)\iff$ $x=z\ \vee\ y(x+z)=xz\iff$ $x=z\ \vee\ \frac 1y=\frac 1x+\frac 1z\iff $ $\boxed{\frac 1y=\frac 1x+\frac 1z}$ . Indeed, if $x=z$ , then the inequality

becomes $2\sqrt {x^2-xy+y^2}\ge x\sqrt 3$ $\iff $ $x^2-4xy+4y^2\ge 0\iff$ $(x-2y)^2\ge 0$ . Obtain equality $\iff$ $\frac x2=\frac y1=\frac z2$ and in this case $\frac 1y=\frac 1x+\frac 1z$ .

Proof 3. $\sqrt{x^2-xy+y^2}+\sqrt{y^2-yz+z^2}\ge \sqrt{x^2+xz+z^2}\iff$

$x^2-xy+y^2+y^2-yz+z^2+2\sqrt{(x^2-xy+y^2)(y^2-yz+z^2)}\geq x^2+xz+z^2\implies $

$2\sqrt{(x^2-xy+y^2)(y^2-yz+z^2)}\geq xy+yz+xz-2y^2\implies$

$ 4(x^2-xy+y^2)(y^2-yz+z^2)\geq (xy+yz+az-2y^2)^2\implies$ $ 3(xy-xz+yz)^2\geq 0$ .


An easy extension. $\left\|\begin{array}{c}
\{x,y,z,m\}\subset\mathbb R\\\\
0\ \le\ m\ \le\ 2\end{array}\right\|\ \implies\ \sqrt{x^2-mxy+y^2}+$ $\sqrt{y^2-myz+z^2}\ge $ $\sqrt{x^2+\left(2-m^2\right)xz+z^2}$ with equality iff $\frac 1x+\frac 1z=\frac my$ .

Proof. Let $\triangle VBA$ , $\triangle VBC$ where $VB$ separates $A$ , $C$ and $\left\{\begin{array}{ccc}
VA=x\ \ \wedge\ \  VB=y\ \ \wedge\ \ VC=z\\\\
m\left(\widehat{AVB}\right)=m\left(\widehat{CVB}\right)=\phi\end{array}\right\|$ , where $\phi\in\left[0,\frac {\pi}{2}\right]$ . Denote $m=2\cos\phi\in [0,2]$

Observe that $2\cos 2\phi =2\left(2\cos^2\phi -1\right)=m^2-2$ and $\left\{\begin{array}{c}
AB^2=x^2-mxy+y^2\\\\
CB^2=z^2-mzy+y^2\\\\
AC^2=x^2+\left(2-m^2\right)xz+z^2\end{array}\right\|$ and our inequality is equivalently with $AB+CB\ge AC$ ,

what is truly. We have equality $\iff B\in (AC)\ \iff$ in $\triangle AVC$ the edge $[VB]$ is the $V$-bisector in $\triangle AVC\iff$

$\frac {BA}{BC}=\frac {VA}{VC}\iff$ $\frac {\sqrt{y^2-myx+x^2}}{y^2-myz+z^2}=\frac xz\iff$ $z^2\left(y^2-myx+x^2\right)=x^2\left(y^2-myz+z^2\right)\iff$ $y^2\left(x^2-z^2\right)=mxyz(x-z)\iff$

$x=z\ \vee\ y(x+z)=mxz\iff$ $x=z\ \vee\ \frac my=\frac 1x+\frac 1z\iff $ $\boxed{\frac my=\frac 1x+\frac 1z}$ .



PP8. Let $a,b,c$ be positive real numbers. Prove that $a^3 +b^3 +c^3\ \geq\ a^2b +b^2c +c^2a$ .

Proof. $\left\{\begin{array}{ccc}
a^3+a^3+b^3 & \ge & 3\cdot a^2b\\\\
b^3+b^3+c^3 & \ge & 3\cdot b^2c\\\\
c^3+c^3+a^3 & \ge & 3\cdot c^2a\end{array}\right\|\ \bigoplus\ \implies\ a^3+b^3+c^3\ \ge\ a^2b+b^2c+c^2a$ .

An easy extension. Let $a,b,c$ be positive real numbers. Prove that $a^{n+1}+b^{n+1} +c^{n+1}
 \geq\ a^nb +b^nc +c^na$ for any $n\in\mathbb N$ .

Proof. $\left\{\begin{array}{ccc}
\underbrace{a^{n+1}+a^{n+1}+\ \ldots\ +a^{n+1}}_n+b^{n+1} & \ge & (n+1)\cdot a^nb\\\\
\underbrace{b^{n+1}+b^{n+1}+\ \ldots\ +b^{n+1}}_n+c^{n+1} & \ge & (n+1)\cdot b^nc\\\\
\underbrace{c^{n+1}+c^{n+1}+\ \ldots\ +c^{n+1}}_n+a^{n+1} & \ge & (n+1)\cdot c^na\end{array}\right\|\ \bigoplus$ $\implies\ a^{n+1}+b^{n+1} +c^{n+1}
 \geq\ a^nb +b^nc +c^na$ for any $n\in\mathbb N$ .


PP9. Let $a,b,c$ be three positive numbers . Prove that $\left\{\begin{array}{cccc}
8(a+b+c)(ab+bc+ca) & \le & 9(b+c)(c+a)(a+b) & (1)\\\\
2\cdot \sqrt{bc+ca+ab} &  \leq & \sqrt{3}\cdot \sqrt[3]{(b+c)(c+a)(a+b)} & (2)\\\\
8\sqrt {abc(a+b+c)(ab+bc+ca)} & \le & 3(b+c)(c+a)(a+b) & (3)\end{array}\right\|$ .

Proof. Denote $\left\{\begin{array}{ccc}
a+b+c & = & s_1\\\
ab+bc+ca & = & s_2\\\
abc & = & s_3\end{array}\right\|$ , i.e. $(t-a)(t-b)(t-c)=t^3-s_1\cdot t^2+s_2\cdot t-s_3=0\begin{array}{cc}
\nearrow & a\\\
\rightarrow &  b\\\
\searrow & c\end{array}$ .

Observe that $s_1s_2=(a+b+c)(ab+bc+ca)\ge$ $ 3\sqrt [3]{abc}\cdot 3\sqrt [3]{a^2b^2c^2}=$ $9abc=9s_3\implies \boxed{s_1s_2\ge 9s_3}\ (*)$ .

Therefore, $(b+c)(c+a)(a+b)=(s_1-a)(s_1-b)(s_1-c)=$ $s_1^3-s_1^2\cdot s_1+s_1s_2-s_3=s_1s_2-s_3\implies$

$9(b+c)(c+a)(a+b)=9s_1s_2-9s_3\stackrel{(*)}{\ge}9s_1s_2-s_1s_2=8s_1s_2\implies$ $\boxed{9(b+c)(c+a)(a+b)\ge 8(a+b+c)(ab+bc+ca)}\ (1)$ .

In conclusion, $\left\{\begin{array}{ccc}
81(b+c)^2(c+a)^2(a+b)^2 & \ge & 64(a+b+c)^2(ab+bc+ca)^2\\\\
(a+b+c)^2 & \ge &  3(ab+bc+ca)\end{array}\right\| \bigodot\ \implies$

$27(b+c)^2(c+a)^2(a+b)^2\ge 64(ab+bc+ca)^3\iff$ $\boxed{\sqrt{3}\cdot \sqrt[3]{(b+c)(c+a)(a+b)}\ge 2\cdot \sqrt{bc+ca+ab}}\ (2)$ .

$\left\{\begin{array}{ccc}
81(b+c)^2(c+a)^2(a+b)^2 & \ge & 64(a+b+c)^2(ab+bc+ca)^2\\\\
(a+b+c)^2 & \ge &  3(ab+bc+ca)\\\\
(ab+bc+ca)^2 & \ge & 3abc(a+b+c)\end{array}\right\| \bigodot\ \implies$

$9(b+c)^2(c+a)^2(a+b)^2\ge 64abc (a+b+c)(ab+bc+ca)\iff$ $\boxed{3(b+c)(c+a)(a+b)\ge 8\cdot \sqrt{abc(a+b+c)bc+ca+ab)}}\ (3)$ .



PP10. Prove that for any $\{a,b,c\}\subset\mathbb R^*_+$ there is the inequality $2\cdot \sum a^6+16\cdot \sum b^3c^3\ge 9\cdot\sum b^2c^2\left(b^2+c^2\right)$ .

Proof. I"ll show that $b^6+c^6+16b^3c^3\ge 9b^2c^2\left(b^2+c^2\right)\ (*)$ . Indeed, using the substitution $\frac bc=t$ the relation $(*)$ becomes $t^6+1+16t^3-9t^2\left(t^2+1\right)\ge 0$ . Using the substitution

$t+\frac 1t=y$ obtain that $t^3+\frac {1}{t^3}+16-9\left(t+\frac 1t\right)\ge 0\iff$ $y^3-3y+16-9y\ge 0\iff$ $y^3-12y+16\ge 0\iff$ $(y-2)^2(y+4)\ge 0$ , what is truly. I have equality

if and only if $y=2\iff t=1\iff b=c$ . In conclusion, $\left\{\begin{array}{c}
b^6+c^6+16b^3c^3\ge 9b^2c^2\left(b^2+c^2\right)\\\\
c^6+a^6+16c^3a^3\ge 9c^2a^2\left(c^2+a^2\right)\\\\
a^6+b^6+16a^3b^3\ge 9a^2b^2\left(a^2+b^2\right)\end{array}\right\|\ \bigoplus\ \implies 2\cdot \sum a^6+$ $16\cdot \sum b^3c^3\ge 9\cdot\sum b^2c^2\left(b^2+c^2\right)$ .



PP11. Find the maximum value of $\left|\sqrt{x^2+4x+5}-\sqrt{x^2+2x+5}\right|^4$ .

Proof 1 $.\ \left|\sqrt{x^2+4x+5}-\sqrt{x^2+2x+5}\right|=\left|\sqrt{(x+2)^2+(0-1)^2}-\sqrt{(x+1)^2+(0-2)^2}\right|$ . Let a mobile

$P(x,0)\in\mathrm{Ox}$ and fixed $A(-2,1)$ , $B(-1,2) $ . Now $\left|\sqrt{x^2+4x+5}-\sqrt{x^2+2x+5}\right|=|PA-PB|\le AB=\sqrt 2$ .

Have the equality for $P\in AB\cap\mathrm{Ox}$ $\iff P(-3,0)$ . Hence $\max \left|\sqrt{x^2+4x+5}-\sqrt{x^2+2x+5}\right|^4= 4 \text{ at  }x=-3$ .



PP12. Prove that $\{x,y,z\}\subset\mathbb R\implies 8\cdot \large{(x^3+y^3+z^3)^2 \ge 9\cdot (x^2+yz)(y^2+zx)(z^2+xy)}$ .

Proof. $27\cdot (x^2+yz)(y^2+zx)(z^2+xy) \le (x^2+y^2+z^2+xy+yz+zx)^3 \le$ $ 8\cdot (x^2+y^2+z^2)^3 \le 24\cdot (x^3+y^3+z^3)^2$ .

Remark. I used the remarkable power main inequality $\sqrt[2]{\frac {x^2+y^2+z^2}{3}}\le \sqrt [3]{\frac {x^3+y^3+z^3}{3}}$ .



PP13. Let $a>0$ , $b>0$ and $a+b=1$ . Prove that $2<\left(a-\frac{1}{a}\right)\left(b-\frac{1}{b}\right)\le\frac{9}{4}$ .

Proof. $\{a,b\}\subset \mathbb R^*_+$ and $a+b=1\implies$ $\left(a-\frac{1}{a}\right)\left(b-\frac{1}{b}\right)=$ $\frac {\left(1-a^2\right)\left(1-b^2\right)}{ab}=$ $\frac {(1+a)(1-a)(1+b)(1-b)}{ab}=$

$\frac {(1+a)b(1+b)a}{ab}=$ $(1+a)(1+b)=1+(a+b)+ab=2+ab$ . In conclusion, $2<2+ab\le 2+\left(\frac {a+b}{2}\right)^2=\frac{9}{4}$ .



PP14. Prove that for $ a_k>0\ (k=1,\ 2,\ \cdots n)$ , $ \sum_{k=1}^n a_k^2+\frac{n(n+1)(2n+1)}{6}\geq 2\sum_{k=1}^n ka_k$ .

Proof. $\sum_{k=1}^n\left(a_k-k\right)^2\ge 0\iff$ $\sum_{k=1}^na_k^2+\sum_{k=1}^nk^2\ge 2\sum_{k=1}^nka_k\iff$ $ \sum_{k=1}^n a_k^2+\frac{n(n+1)(2n+1)}{6}\geq 2\sum_{k=1}^n ka_k$ .


PP15. Let $\in\left(0,\frac{\pi}{2}\right) $ . Prove that $\frac{3\sqrt{3}}{\sin x}+\frac{1}{\cos x}\ge 8$ .

Proof 1. Observe that $2\left(\frac {3\sqrt 3}{\sin x}+\frac 1{\cos x}\right)\ge \left(\sqrt 3\sin x+\cos x\right)\left(\frac {3\sqrt 3}{\sin x}+\frac 1{\cos x}\right)=$

$\left(\frac {\sqrt 3}{\sin x}+\frac {\sqrt 3}{\sin x}+\frac {\sqrt 3}{\sin x}+\frac 1{\cos x}\right)\left(\frac {\sin x}{\sqrt 3}+\frac {\sin x}{\sqrt 3}+\frac {\sin x}{\sqrt 3}+\cos x\right)\ge 4^2=16$ $\implies$ $\frac{3\sqrt{3}}{\sin x}+\frac{1}{\cos x}\ge 8$ .

Proof 2 (ugly). Observe that $\frac {3\sqrt 3}{\sin x}+\frac 1{\cos x}\ge 8\iff$ $\left(\sin^2x+\cos^2x\right)\left(3\sqrt 3\cos x+\sin x\right)^2\ge$ $64\sin^2x\cos^2x\ \stackrel{\tan x=t}{\iff}$ $\left(t^2+1\right)$ $\left(t+3\sqrt 3\right)^2\ge 64t^2\iff$

$t^4+6\sqrt 3t^3-36t^2+6\sqrt 3t+27\ge 0\iff$ $\left(t-\sqrt 3\right)^2\left(t^2+8\sqrt 3t+9\right)\ge 0$ , what is truly. We have the equality if and only if $t=\sqrt 3$ , i.e. $\tan x=\sqrt 3\iff x=\frac {\pi}{3}$ .

Proof 3. $\frac{3\sqrt{3}}{\sin x}+\frac{1}{\cos x}=$ $\frac{\sqrt{3}}{\sin x}+\frac{\sqrt{3}}{\sin x}+\frac{\sqrt{3}}{\sin x}+\frac{1}{\cos x}\ge $ $4\cdot\sqrt[4]{\frac {3\sqrt3\left(\sin^2x+\cos^2x\right)^2}{\sin^3x\cos x}}$ $\stackrel{(\tan x=t)}{=}$ $4\cdot\sqrt[4]{\frac {3\sqrt3\left(t^2+1\right)^2}{t^3}}$ . Therefore, $\frac{3\sqrt{3}}{\sin x}+\frac{1}{\cos x}\ge 8\iff$

$3\sqrt 3\left(t^2+1\right)^2\ge 16t^3\iff$ $3\sqrt 3t^4-16t^3+6\sqrt 3t^2+3\sqrt 3\ge 0\iff$ $\left(t-\sqrt 3\right)^2\left(3\sqrt 3t^2+2t+\sqrt 3\right)\ge 0$ , it is true. Have equality iff $\tan x=t=\sqrt 3$ , i.e. $x=60^{\circ}$ .


An easy extension (sqing). Prove that for any $\{a,b,u,v\}\in \mathbb R^*_+$ there is the inequality $\boxed{\frac {a}{u}+\frac b{v}\ge \frac {(a+b)^2}{\sqrt {\left(a^2+b^2\right)\left(u^2+v^2\right)}}}$ .

Proof. $\frac au+\frac bv=\frac {a^2}{au}+$ $\frac {b^2}{bv}\ \stackrel{C.B.S}{\ge}\ \frac {(a+b)^2}{au+bv}\ \stackrel{C.B.S}{\ge}\ \frac {(a+b)^2}{\sqrt {\left(a^2+b^2\right)\left(u^2+v^2\right)}}$ . Particular case. If $\{a,b,u,v\}\in \mathbb R^*_+$ and $x\in\left(0,\frac {\pi}{2}\right)$ , then $\frac {a^2}{u\sin x}+\frac {b^2}{v\cos x}\ge \frac{(a+b)^2}{\sqrt {u^2+v^2}}$ .


PP16. Prove that the implication: $x\in\left(0,\pi\right)\ \implies\ \sin x(1-\cos x)\le\frac {3\sqrt 3}{4}$ .

Proof 0 (with derivatives). Denote $f(x)=\sin x(1-\cos x)\ ,\ x\in (0,\pi )$ . Prove easily that $f'(x)=-2\cos^2x+\cos x+1$ $\iff$ $\boxed{f'(x)=(1-\cos x)(2\cos x+1)}$

and $f'(x)=0\iff x=\frac {2\pi}{3}\in (0,\pi)$ . In conclusion $,\ \left\|\begin{array}{c}
x\\\\
f'\\\\
f\end{array}\right|$ $\left|\begin{array}{ccc}
& \frac {2\pi}{3} & \\\\
+ & 0 & -\\\\
\nearrow & \frac {3\sqrt 3}{4} & \searrow\end{array}\right\|\implies$ $\boxed{\max_{0<x<\pi}f(x)=f\left(\frac {2\pi}{3}\right)=\frac {3\sqrt 3}{4}}$ , i.e. $x\in\left(0,\pi\right)\implies\sin x(1-\cos x)\le\frac {3\sqrt 3}{4}$ .

Proof 1. $\sin x(1-\cos x)$ is maximum $\iff$ $\sin^2x(1-\cos x)^2$ is maximum $\iff$ $(1+\cos x)(1-\cos x)^3$ is maximum $\iff$ $(1+\cos x)\cdot \frac {1-\cos x}{3}\cdot\frac {1-\cos x}{3}\cdot\frac {1-\cos x}{3}$

is maximum . Since $(1+\cos x)+\frac {1-\cos x}{3}+\frac {1-\cos x}{3}+\frac {1-\cos x}{3}=2$ (constant) obtain that $\sin x(1-\cos x)$ is maximum $\iff$ $1+\cos x=\frac {1-\cos x}{3}=\frac 24\iff$

$\cos x=-\frac 12\iff$ $x=\frac {2\pi}{3}$ . With othet words, $\sin x(1-\cos x)\le \sin\frac {2\pi}{3}\left(1-\cos \frac {2\pi}{3}\right)\iff$ $\sin x(1-\cos x)\le\frac {3\sqrt 3}{4}$ .

Proof 2. $\sin x(1-\cos x)=4\sin^3\frac x2\cos\frac x2=$ $\frac {4\sin^3\frac x2\cos\frac x2}{\left(\sin^2\frac x2+\cos^2\frac x2\right)^2}=$ $\frac {4t^3}{\left(t^2+1\right)^2}$ , where $t=\tan\frac x2\in(0,\infty)$ . Thus, $\sin x(1-\cos x)\le\frac {3\sqrt 3}{4}\iff$

$3\sqrt 3\left(t^2+1\right)^2-16t^3\ge 0\iff$ $\left(t-\sqrt 3\right)^2\left(3\sqrt 3t^2+2t+\sqrt 3\right)\ge 0$ , what is truly. Have equality if and only if $t=\sqrt 3\iff$ $\tan\frac x2=\sqrt 3\iff$ $x=\frac {2\pi}{3}$ .


An easy extension. Prove that the implication: $\{m,n\}\subset\mathbb N^*\ ,\ x\in\left(0,\pi\right)\ \implies\ \sin^m x(1-\cos x)^n\le\frac {(m+2n)^n\sqrt {m(m+2n)}}{(m+n)^{n+1}}$ .
This post has been edited 157 times. Last edited by Virgil Nicula, Nov 21, 2015, 5:43 PM

Comment

0 Comments

Own problems or extensions/generalizations of some problems which was posted here.

avatar

Virgil Nicula
Archives
+ October 2017
+ September 2017
+ December 2016
+ October 2016
+ February 2016
+ September 2013
+ October 2010
+ September 2010
Shouts
Submit
  • orzzzzzzzzz

    by mathMagicOPS, Jan 9, 2025, 3:40 AM

  • this css is sus

    by ihatemath123, Aug 14, 2024, 1:53 AM

  • 391345 views moment

    by ryanbear, May 9, 2023, 6:10 AM

  • We need virgil nicula to return to aops, this blog is top 10 all time.

    by OlympusHero, Sep 14, 2022, 4:44 AM

  • :omighty: blog

    by tigerzhang, Aug 1, 2021, 12:02 AM

  • Amazing blog.

    by OlympusHero, May 13, 2021, 10:23 PM

  • the visits tho

    by GoogleNebula, Apr 14, 2021, 5:25 AM

  • Bro this blog is ripped

    by samrocksnature, Apr 14, 2021, 5:16 AM

  • Holy- Darn this is good. shame it's inactive now

    by the_mathmagician, Jan 17, 2021, 7:43 PM

  • godly blog. opopop

    by OlympusHero, Dec 30, 2020, 6:08 PM

  • long blog

    by MrMustache, Nov 11, 2020, 4:52 PM

  • 372554 views!

    by mrmath0720, Sep 28, 2020, 1:11 AM

  • wow... i am lost.

    369302 views!

    -piphi

    by piphi, Jun 10, 2020, 11:44 PM

  • That was a lot! But, really good solutions and format! Nice blog!!!! :)

    by CSPAL, May 27, 2020, 4:17 PM

  • impressive :D
    awesome. 358,000 visits?????

    by OlympusHero, May 14, 2020, 8:43 PM

72 shouts
Tags
About Owner
  • Posts: 7054
  • Joined: Jun 22, 2005
Blog Stats
  • Blog created: Apr 20, 2010
  • Total entries: 456
  • Total visits: 404427
  • Total comments: 37
Search Blog
a